Bytelearn - cat image with glassesAI tutor

Welcome to Bytelearn!

Let’s check out your problem:

z=-3+5i
Find the angle 
theta (in radians) that 
z makes in the complex plane. Round your answer, if necessary, to the nearest thousandth. Express 
theta between 
-pi and 
pi.

theta=

z=3+5i z=-3+5 i \newlineFind the angle θ \theta (in radians) that z z makes in the complex plane. Round your answer, if necessary, to the nearest thousandth. Express θ \theta between π -\pi and π \pi .\newlineθ= \theta=

Full solution

Q. z=3+5i z=-3+5 i \newlineFind the angle θ \theta (in radians) that z z makes in the complex plane. Round your answer, if necessary, to the nearest thousandth. Express θ \theta between π -\pi and π \pi .\newlineθ= \theta=
  1. Calculate Argument: To find the angle θ\theta that the complex number z=3+5iz = -3 + 5i makes with the positive real axis in the complex plane, we need to calculate the argument of the complex number. The argument is the angle in polar coordinates and can be found using the arctan\text{arctan} function, which gives the angle whose tangent is the quotient of the imaginary part over the real part of the complex number.
  2. Find Tangent Ratio: The complex number z=3+5iz = -3 + 5i has a real part of 3-3 and an imaginary part of 55. The tangent of the angle θ\theta is the ratio of the imaginary part to the real part, which is 53\frac{5}{-3}. Therefore, we use the arctan\text{arctan} function to find the angle: θ=arctan(53)\theta = \text{arctan}(\frac{5}{-3}).
  3. Adjust for Quadrant: However, since the real part of zz is negative and the imaginary part is positive, zz lies in the second quadrant of the complex plane. The range of the arctan\text{arctan} function is (π2,π2)(-\frac{\pi}{2}, \frac{\pi}{2}), but we need to adjust the angle because the arctan\text{arctan} function does not account for the quadrant in which the complex number lies. To find the correct angle in the second quadrant, we add π\pi to the arctan\text{arctan} value.
  4. Calculate Arctan Value: Calculating the arctan value: θ=arctan(53)=arctan(53)\theta = \text{arctan}(\frac{5}{-3}) = \text{arctan}(-\frac{5}{3}). Using a calculator, we find that arctan(53)1.0304\text{arctan}(-\frac{5}{3}) \approx -1.0304 radians. Since this is in the fourth quadrant and we need the angle in the second quadrant, we add π\pi to get the correct angle: θ=1.0304+π\theta = -1.0304 + \pi.
  5. Add Pi: Adding π\pi to 1.0304-1.0304 radians, we get θ1.0304+3.1416=2.1112\theta \approx -1.0304 + 3.1416 = 2.1112 radians. Since we want the angle to be expressed between π-\pi and π\pi, and 2.11122.1112 radians is within this range, this is our final answer.

More problems from Find trigonometric ratios using a Pythagorean or reciprocal identity